For which value of 0 is 0=-1 ?

For Which Value Of 0 Is 0=-1 ?

Answers

Answer 1

Answer:

[tex]theta = \frac{3}{2} \pi + 2k\pi[/tex]

Step-by-step explanation:

+2*k*pi is for the periodicity (if you turn 360° you get back to the same point)


Related Questions

Find the angle marked with the ? mark

Answers

Answer:

53 degrees

Step-by-step explanation:

Angle N = angle E

because angle made by joining end points of same chord on circumference are always equal.

so angle E = 37

Angle D = 90 ( because angle made by diameter on circumference is 90 degrees)

Now in Triangle DEC. Sum if all the angles of triangle us 180

Angle D + angle E + ? = 180

37 + 90 + ? = 180

127 + ? = 180

? = 180 - 127

? = 53 degrees

Find the area of the triangle bounded by the lines y=x y=-x and y=6.

Answers

S=6*12=72

High of triangle is 6

The area of the triangle bounded by the lines y=x y=-x and y=6 is 36 units.

What is area of triangle?

The formula for finding area could be represented in the form of determinants as given below.

[tex]A = \frac{1}{2} \left[\begin{array}{ccc}x1&y1&1\\x2&y2&1\\x3&y3&1\end{array}\right][/tex]

First, we need to find the coordinates of the point of intersection of these lines.

y = x

y = -x

Adding the two equations,

2y = 0

y = 0

x = 0

coordinate: (0, 0)

y = x

y = 6

Subtracting the two equations,

0 = x - 6

x = 6

coordinate: (6, 6)

y = -x

y = 6

Subtracting the two equations,

- x - 6 = 0

x = -6

coordinate: (-6, 6)

Calculating area of triangle bounded by the given line:

Area of triangle =

[tex]\frac{1}{2}\left[\begin{array}{ccc}0&0&1\\6&6&1\\-6&6&1\end{array}\right][/tex]  =  [tex]\frac{1}{2} (36 + 36) = \frac{72}{2} = 36[/tex]

Learn more about area of triangle here

https://brainly.com/question/19305981

#SPJ2

Show that x +1 is a factor of f(x) =2x^3 +3x^2 - 5x - 6

Answers

Answer:

Step-by-step explanation:

To solve this, we can use polynomial long division.

Seeing the picture, we first divide x (the variable to the largest degree in x+1) from 2x³. We divide from 2x³ because that is the variable with the largest degree in the polynomial. That is equal to 2x², so we put that on top and subtract (x+1) * (2x²) from the polynomial. Then, we repeat the process, but with x² instead of 2x³, and again with -6x as the variable with the largest degree.

WORTH 25 POINTS!!!!!! PLS PLS PLS PLS PLS PLS PLS HELP I DON'T GET IT!!! Write missing monomials to make an identity:
A) (.....+2a)^2=.....+12ab+4*....


B) (3x+.....)^2=....*x^2+.....+49y^2

Answers

Answer:

(3x + 7y)^2 =  9 x^2 + 42xy + 49y^2

Step-by-step explanation:

Find f(2) if f(x) = (x + 1)^2.
9
6
5

Answers

Answer:

9

Step-by-step explanation:

f(2)=(2+1)^2

(3)^2

(9)

divide 3 divided by 2/5

Answers

Answer:

[tex]{ \tt{ = 3 \div \frac{2}{5} }} \\ = { \tt{3 \times \frac{5}{2} }} \\ = \frac{15}{2} [/tex]

9. Determine the volume of concrete needed to build a ramp in the shape of a triangular prism to the nearest tenth of a cubic metre. ​

Answers

Answer:

67.5

Step-by-step explanation:

I've learned this multiple ways but in my opinion this is the easiest. Just times everything together so, 40 x 2.5 x 1.2 which equals 135 and then divide 135 by 2 which equals your answer of 67.5 which is rounded to the nearestt tenth already.

What's 70 x4 I just wanna see if it’s really going to help me

Answers

Step-by-step explanation:

70

*4

___

280.......ans

An x-method chart shows the product a c at the top of x and b at the bottom of x. Below the chart is the expression a x squared + b x + c. What are the factors of x2 – 144? And

Answers

Answer:

[tex](x -12)(x + 12)[/tex]

Step-by-step explanation:

Given

See attachment for chart

Required

The factors of [tex]x^2 - 144[/tex]

First, express [tex]x^2 - 144[/tex] as [tex]ax^2 + bx + c[/tex]

So, we have:

[tex]x^2 - 144 = x^2 + 0x - 144[/tex]

Compare the above expression to: [tex]ax^2 + bx + c[/tex]

We have:

[tex]ax^2 + bx + c = x^2 + 0x - 144[/tex]

So:

[tex]a =1[/tex]

[tex]b =0[/tex]

[tex]c = -144[/tex]

and

[tex]a * c = d * e[/tex]

Calculate ac

[tex]a* c = 1 * -144[/tex]

[tex]a* c = -144[/tex]

Rewrite as:

[tex]a* c = -12 * 12[/tex]

Recall that:

[tex]a * c = d * e[/tex]

Hence:

[tex]d = -12; e = 12[/tex]

So, on the x chart, we have:

        ac

d                  e

        b

This gives:

        -144

-12                  12

        0

The factors are

[tex](x + d)(x + e)[/tex]

[tex](x -12)(x + 12)[/tex]

Answer:

✔ (x – 12)

 and  

✔ (x + 12)

Step-by-step explanation:

Solve for x.
3х - 4 = 13

Answers

[tex]\large\bold{\underline{\underline{ \: x = 5 \frac{2}{3} }}}[/tex]

[tex]\large\mathfrak{{\pmb{\underline{\orange{Step-by-step\:explanation}}{\orange{:}}}}}[/tex]

[tex]3x - 4 = 13[/tex]

[tex]➺ \: 3x = 13 + 4[/tex]

[tex]➺ \: 3x = 17[/tex]

[tex]➺ \: x = \frac{ 17}{3} [/tex]

[tex]➺ \: x = 5 \frac{2}{3} [/tex]

[tex]\large\mathfrak{{\pmb{\underline{\blue{To\:verify}}{\blue{:}}}}}[/tex]

[tex]➺ \: 3x - 4 = 13[/tex]

[tex]➺ \: 3 \times 5 \frac{2}{3} - 4 = 13[/tex]

[tex]➺ \: 3 \times \frac{17}{3} - 4 = 13[/tex]

[tex]➺ \: 17 - 4 = 13[/tex]

[tex]➺ \: 13 = 13[/tex]

➺ L. H. S. = R. H. S.

Hence verified.

[tex]\red{\large\qquad \qquad \underline{ \pmb{{ \mathbb{ \maltese \: \: Mystique35♛}}}}}[/tex]

Answer: X = 6
Hope it helps:)

Which point is in the solution set of this system inequalities?

A. (0,0)

B. None of these

C. (5,1)

D. (3,7)

Answers

Answer:

B

Step-by-step explanation:

To find which ordered pairs are solutions to the inequalities we can simply plug in the x and y values of the ordered pairs into the inequalities and if the equation is true for both inequalities then the ordered pair is a solution to the inequalities.

For (0,0)

x = 0

y = 0

y > x + 5

Substitute 0 for y and x

0 > 0 + 5

Simplify right side

0 > 5

The inequality is not true as 5 is greater than 0, not less than. So immediately we can eliminate answer choice A.

For (5,1).

x = 5

y = 1

y > x + 5

Substitute 5 for x and 1 for y

1 > 5 + 5

Simplify right side

1 > 10

Again, the equation is not true as 1 is not greater than 10. This means that c cannot be the answer

For (3,7)

x = 3

y = 7

y > x + 5

Substitute 3 for x and y for 7

7 > 3 + 5

Simplify right side

7 > 8

7 is not greater than 8 meaning that (3,7) cannot be a solution to the inequalities

None of the ordered pairs created true equations hence the answer is B

Here is the distribution of blood types from a group
of randomly selected people:
o
А
B
AB
Blood
Type:
Probability:
0.49
0.27
0.20
?
What is the probability of type AB blood?

Answers

0.04 is the probability of type AB blood

b) In a survey of 200 families, 80 were found using 'A brand ton only, 75 were found using 'B' brand tea only and each family is using at least one of the two brands, (1) Draw a Venn diagram to illustrate the above information (i) How many families are using both brande? (111) How many families are using 'A' brand? (iv) How many families are using 'B' brand?​

Answers

Answer:

The answer would be D

Step-by-step explanation:

I did the quiz

Step-by-step explanation:

n(u)=200

no(A)=80

no(B)=75

ii) Using both brand=200-80-75

=45

iii)n(A)=80+45

=125

iv)n(B)=75+45

=120

Stephanie found a sweater on sale for $12.88. The original price of the sweater was $44.95. About how much did she save on the sweater?
rounding

Answers

Answer:

$32.07

ROUNDING ANSWER : $32

Step-by-step explanation:

you would subtract $44.95 -$12.88 which would give you $32.07  

since you are rounding your answer would be $32

Hope this helps

Answer:

Stephanie saved about $32 on her new sweater.

$44.95 - $12.88 = $32.07

$32.07 rounds down to $32

Step-by-step explanation:

Have the same assignment. Hope this helps!

Which are correct Representation of the any quality 6x>_3+4(2x-1)?select three options

Answers

Answer:

1) 1 ≥ 2·x

2) 6·x ≥ 3 + 8·x - 4

3) 1/2 ≥ x The third option and the first number line inequality diagram, please see attached drawing of the number line created with MS Visio

Step-by-step explanation:

The given inequality is presented as follows;

6·x ≥ 3 + 4·(2·x - 1)

By expanding the right hand side of the inequality, we get;

6·x ≥ 3 + 8·x - 4

4 - 3 = 1 ≥ 8·x - 6·x = 2·x

∴ 1 ≥ 2·x

1/2 ≥ x

Therefore, the correct options are;

1) 1 ≥ 2·x

2) 6·x ≥ 3 + 8·x - 4

3) 1/2 ≥ x The third option and the first number line inequality diagram

A condition statement is logically equivalent to a biconditional statement. true or false​

Answers

Answer:

true

Step-by-step explanation:

Because a logically equivalent is the same as biconditional statement

Step-by-step explanation:

hello the answer is true, you can check but it's obviously true

Factorise 2ab+2ac. Please

Answers

Answer:

2a(b+c)

Step-by-step explanation:

Find HCF (2a)

Then factorise

Answer:

2a(b+c)

.............

add: -38+6+27+(-8)+126

Answers

Answer:

113

Step-by-step explanation:

How can I get the answer

Answers

Answer:

A.

Step-by-step explanation:

To find the inverse of a function, first make f(x) a y

So,

y = [tex]\frac{12}{x}[/tex] -18

Then switch the x and the y

x = [tex]\frac{12}{y}[/tex] - 18

Solve, for y

(-9g5+ 9) + (4g5- 5)

Answers


−9​g5+9+4g5−5

−9g5+4g5+9−5

−5g5+9−5

Answer: −5g5+4



someone help me please with this algebra problem

Answers

Answer:

7 in.

Step-by-step explanation:

2L + 2W = 16

Try L = 7

2(7) + 2W = 16

14 + 2W = 16

2W = 2

W = 1

If the length is 7 in., then the width is 1 in. That is perfectly acceptable, so L = 7 in. is a good value.

Try L = 8

2(8) + 2W = 16

16 + 2W = 16

2W = 0

W = 0

A length of 8 would make a width of 0. You can't have a rectangle with 0 width, so L = 8 does not work.

When L = 9 or L = 10, the width would be negative. The width of a rectangle cannot be a negative number, so these values doe not work.

Answer: 7 in.

Look at the graph below. What type of function is represented by this graph?

Someone come through with the answers pls

Answers

So you need to get your slope formed then rise over run and calculate your graph. Y=Mx+b

Tickets to a football final are selling well. On Thursday, 47 of the tickets are sold. On Friday, 14 of the tickets are sold. What fraction of tickets are available to sell on Saturday?

Answers

The question seems incomplete ; as the total number of tickets to be sold isn't given.

Answer:

61 / X

Step-by-step explanation:

Let's take the total Number of tickets to be sold as : X

Number of tickets sold on Thursday = 47

Number sold on Friday = 14

Fraction of tickets available for sale on Saturday :

(Total number of tickets already sold) / Total number of tickets to be sold

(Thursday + Friday sales) / total number of tickets to be sold

Fraction available for sale on Saturday = (47+14) / X

Fraction available for sale on Saturday = 61 / X

Kindly put value of x = total number of tickets available for sale to get the exact fraction.

Pls help ASAP!!!!!

Find the average rate of change from d=4
to d=11 for the function f(d) = 5(1.02)^d. Describe the process and steps he used and explain what the average rate of change represents.

Answers

Answer:

0. 116

Step-by-step explanation:

The function is given as :-

[tex]\boxed{f(d) = 5(1.02)^d }[/tex]

and we have to find the rate of change from d = 4 to d = 11

[tex]\boxed{\blue{\mathfrak{Rate\: of\: change = \frac{final\:output-intital\:output}{final\:input-initial\:input} } }}[/tex]

The final input value is 11 whereas the initial input value is 4.

The final and initial outputs can be calculated by placing the respective values of initial and final inputs (that are 4 and 11).

[tex]{\underline{Initial\:Output}}[/tex]

f(4) = [tex]5(1.02)^4[/tex]

f(4) = 5 × 1. 08

f(4) = 5. 41

[tex]{\underline{Final\:Output}}[/tex]

f(11) = [tex]5(1.02)^11[/tex]

f(11) = 5 × 1. 24

f(11) = 6. 22

[tex]\underline{Avg\:Rate \: of \: change} = \frac{6. 22-5.41}{11-4} \\ = \frac{0.81}{7} \\ = 0.116 [/tex]

[tex]\bigstar[/tex] Hence, the average rate of change is [tex]\red{\underline{\pmb{0. 116}}}[/tex]


If f(x) is an exponential function where f(-1.5)=26 and f(5.5)=7 then find the value of f(10), to the nearest hundredth

Answers

Answer:

5.0956537e-46

Step-by-step explanation:

^ means root

* means multiply

f(x) = ab^x

f(-1.5)=26 = ab^-1.5

f(5.5)=7 = ab^5.5

f(5.5)/f(-1.5) = 26/7 = ab^5.5/ab^-1.5

3.714 = b^7

b = 3.714^-7

f(-1.5)=26 = ab^-1.5

26 = ab^-1.5

26 = a * 3.714^-7

a = 3.714^-7 / 26

a = 0.00000394578

f(10) = ab^10

f(10) = 0.00000394578 * (3.714^-7)^10

f(10) = 5.0956537e-46

A bag contains 6 black tiles, 5 white tiles, and 4 blue tiles. Event A is defined as drawing a white tile from the bag on the first draw, and event B is defined as drawing a black tile on the second draw. If two tiles are drawn from the bag, one after the other without replacement, what is P(A and B) expressed in simplest form? A. 4/45 B. 1/7 C. 4/15 D. 5/14

Answers

Answer:

5/14

Step-by-step explanation:

There are 15 tiles in total

5 white| 6 black | 4 blue

event A results in the subject pulling a white tile and not replacing it

5-1= 4

so the first answer should be 4/15

event B results in the subject pulling another tile, a black one and not replacing it.

6-1= 5

given this answer, there is one less tile in the total, since we removed another tile.

So our answer would be-

5/14 or D

SOMEONE PLEASE HELP ME OUT!!!!

Answers

Answer:

40/30

Step-by-step explanation:

Since tan∅= o/a, 40 is opposite, and 30 is adjacent to angle A, 40/30 is the ratio for tanA.

Answer:40/30

Step-by-step explanation:

....

In right ΔDEF, DF = 20, m∠ F = 90˚, EF = 17. Which of the following is true? Select all that apply

Answers

Answer:

option 1

Step-by-step explanation:

I am not sure, what your are asking about.

in your text you define the length of DF (20), but the answer options (only 2 visible) also specify DF but differently.

so ... ?

when you said DF=20, did you actually mean DE ?

under that assumption

DE = 20 = Hypotenuse of the right-angled triangle (the opposite side of the 90 degree angle).

EF = 17

based on Pythagoras

c² = a² + b²

we have here now

20² = 17² + DF²

400 = 289 + DF²

DF² = 400 - 289 = 111

DF = sqrt(111)

on a coordinate gride what is the distance between (1,3) and (6,15)

Answers

Answer:

13

Step-by-step explanation:

Distance between points (1, 3) and (6, 15) is 13

13 is the right answer for sure

the red line below is perpendicular to which of the following

Answers

The red line is perpendicular to the y-axis.

Other Questions
Prove that the square of an odd number is always 1 more than a multiple of 4 A recent national study about the effectiveness of Echinacea in cold treatments for adults was performed by a medical school in Kansas City. The results stated that 22.5% of the randomly chosen 250 adult subjects in the placebo group in the study noted that their treatment appeared to shorten the length of their colds. In an attempt to determine the average high school GPA of all students enrolled at a Regents University in Kansas, a researcher first randomly selects one of the six Regents Universities, then selects a random sample of 50 students from that University from which to gather data. The descriptive statistic of interest in this study is 1286Find the value of x.A) 9B) 16C) 14D) 10 A display case of toy rings are marked 5 for $1. If Zach wants to buy 50 toy rings, how much will Zach spend (not including tax) Does anyone know what graph is correct? Whats the main reason why cell specialization is important Misty Copeland is the second African American to reach the level of soloist in the American Ballet Theater (ABT). A woman of color had not been soloist in the ABT since 20 years. Q. 1: How do you think Misty got the high position of soloist in the prestigious ABT? Develop your answer in 1-2 sentences. Use your academic language. what is the answer to this Solve for X (line a and b parallel) A family with an income of 70,000 receives a raise of 2% in a year when inflation is 5%. Find the decrease in purchasing power. write a paragraph on how society come into existence I was stuck by her good looks.From Passive to Active voice Natural selection is most likely to be the cause of a change in a population'sallele frequencies when:A. the ecosystem experiences few disturbances,B. the population drops below its carrying capacity.C. the predators are removed from the ecosystem,D. the population has reached its carrying capacity.PLEASE HELP URGENT What factors differentiate Earth's atmospheric layers?A. Pressure and Rate of condensationB. Radiation deflection and Rate of condensationC. Temperature and PressureD. Temperature and Radiation deflectionPlz help Why is it that although Nocardia spp. grow well on most common nonselective laboratory media, many are missed when the cultures are read PLZ HELPPP!!! whoever answers first with a good answer ill mark as brainliest Service Pro Corp (SPC) is preparing adjustments for its September 30 year- end. For the following transactions and events, show the September 30 adjusting entries that SPC would makea. Prepaid Insurance shows a balance of zero at September 30, but Insurance Expense shows a debit balance of $2,340, representing the cost of a three-year fire insurance policy purchased on September 1 of the current year.b. On August 31 of this year, Cash was debited and Service Revenue was credited for $ 1,500. The $ 1,500 related to fees for a three- month period beginning September 1 of the current year.c. The companys income tax rate is 20%. After making the above adjustments, SPCs net income before tax is $ 10,000. No income tax has been paid or recorded. Which greenhouse gas has an average lifetime in the atmosphere of a few weeks to thousands of years and is the largest contributor to climate change? To Kill a Mockingbird Journaling ProjectFollow the assignment instructions and review the rubric:For this project, you must choose a character from the book and create a journal as if you are that character.Your project must include the following:Eight journal entries of at least a page each. Write these entries from different times along the plot of the story.In four of the journal entries describe a meaningful artifacts that the character might have saved to include with the journal.Design an artistic cover that illustrates the symbols from the book associated with the character.Draw 3 designs or pictures to represent events from the story and note what journal entry they relate to.Describe a creative container that the character might have stored the diary in, for safekeeping.Rubric:To Kill a Mocking Bird Journal AssignmentJournal EntriesEight character journal entries of at least half a page each.Maximum score48TimelineEach one is from different times along the plot of the story.Maximum score16Meaningful ArtifactsDescriptions of four meaningful artifacts that the character might have saved are included within the journal.Maximum score8Cover IllustrationAn artistic cover that illustrates the symbols from the book associated with the character.Maximum score5Designs or PicturesThree designs or pictures to represent events from the story are included, with a title that clearly relates them to a journal entry.Maximum score12Container for the JournalA description of a creative container that the character might have stored the diary in, for safekeeping.Maximum score6GrammarLess than 5 grammatical errors in all journal entriesMaximum score5 what is the answer to this equation45 + v =4120